guide:Ad2677c8f0: Difference between revisions

From Stochiki
No edit summary
 
No edit summary
 
Line 1: Line 1:
<div class="d-none"><math>
\newcommand{\R}{\mathbb{R}}
\newcommand{\A}{\mathcal{A}}
\newcommand{\B}{\mathcal{B}}
\newcommand{\N}{\mathbb{N}}
\newcommand{\C}{\mathbb{C}}
\newcommand{\Rbar}{\overline{\mathbb{R}}}
\newcommand{\Bbar}{\overline{\mathcal{B}}}
\newcommand{\Q}{\mathbb{Q}}
\newcommand{\E}{\mathbb{E}}
\newcommand{\p}{\mathbb{P}}
\newcommand{\one}{\mathds{1}}
\newcommand{\0}{\mathcal{O}}
\newcommand{\mat}{\textnormal{Mat}}
\newcommand{\sign}{\textnormal{sign}}
\newcommand{\CP}{\mathcal{P}}
\newcommand{\CT}{\mathcal{T}}
\newcommand{\CY}{\mathcal{Y}}
\newcommand{\F}{\mathcal{F}}
\newcommand{\mathds}{\mathbb}</math></div>
A very important notion is that of a monotone class. We will see that there are many things which can be deduced by using the ''monotone class lemma''.
{{definitioncard|Monotone Class|
Let <math>E</math> be some topological space and let <math>M\subset\mathcal{P}(E)</math>. <math>M</math> is called a monotone class if the following holds.
<ul style{{=}}"list-style-type:lower-roman"><li><math>E\in M</math>.
</li>
<li>Let <math>A\in M</math> and <math>B\in M</math>. If <math>A\subset B</math> <math>\Longrightarrow B\setminus A\in M</math>.
</li>
<li>Let <math>(A_n)_{n\in\N}\in M</math>. If <math>A_n\subset A_{n+1}</math> <math>\Longrightarrow \bigcup_{n\in\mathbb{N}}A_n \in M</math>.
</li>
</ul>}}
{{alert-info |
A <math>\sigma</math>-Algebra is a monotone class{{efn|Prove that as an exercise.}}
}}
{{alert-info |
As for <math>\sigma</math>-Algebras, we notice that an arbitrary intersection of monotone classes is again a monotone class. Thus, if <math>\mathcal{C}\in \mathcal{P}(E)</math>, we can define the monotone class generated by <math>\mathcal{C}</math> as


<math display="block">
M(\mathcal{C})=\bigcap_{\mathcal{C}\subset M\atop M\text{mon.cl.}}M.
</math>
This is also by construction the smallest monotone class containing <math>\mathcal{C}</math>.
}}
{{proofcard|Theorem (Monotone Classes lemma)|thm-1|Let <math>E</math> be a topological space. If <math>\mathcal{C}\subset\mathcal{P}(E)</math> is stable under finite intersection, i.e. for <math>A\in \mathcal{C}</math> and <math>B\in\mathcal{C}\Longrightarrow A\cap B\in \mathcal{C}</math>, then
<math display="block">
\sigma(\mathcal{C})=M(\mathcal{C}).
</math>
|It is obvious that, <math>M(\mathcal{C})\subset\sigma(\mathcal{C})</math> since a <math>\sigma</math>-Algebra is also a monotone class. Next we want to show that <math>M(\mathcal{C})</math> is a <math>\sigma</math>-Algebra to conclude that <math>\sigma(\mathcal{C})\subset M(\mathcal{C})</math> and hence then <math>M(\mathcal{C})</math> contains <math>\mathcal{C}</math>, i.e. <math>\sigma(\mathcal{C})</math>. It is  not difficult to see that a monotone class, which is stable under finite intersection, is a <math>\sigma</math>-Algebra. Let us therefore show that <math>M(\mathcal{C})</math> is stable under finite intersections. First, we fix <math>A\in\mathcal{C}</math> and define
<math display="block">
M_A:=\{B\in\mathcal{C}\mid A\cap B\in M(\mathcal{C})\}.
</math>
Then we get that <math>\mathcal{C}\in M_A</math> since <math>\mathcal{C}</math> is stable under finite intersections and obviously <math>E\in M_A</math>. We can also note that If <math>B,B'\in M_A</math> and <math>B\subset B'</math>, with
<math display="block">
A\cap (B'\setminus B)=\underbrace{\underbrace{(A\cap B')}_{\in M(\mathcal{C})}\setminus \underbrace{(A\cap B)}_{\in M(\mathcal{C})}}_{\in M(\mathcal{C})}
</math>
then <math>B'\setminus B\in M_A</math>. Moreover, if <math>(B_n)_{n\in\N}\in M_A</math> and <math>A\cap\bigcup_n\underbrace{(A\cap B_n)}_{\in M(\mathcal{C})}</math> we get the implication
<math display="block">
\bigcup_{n\geq 1}(A\cap B_n)\in M(\mathcal{C})\Longrightarrow \bigcup_{n\geq 1}B_n\in M_A.
</math>
since <math>(A\cap B_n)</math> is increasing. Finally we can conclude the above facts. That means if <math>M_A</math> is a monotone class containing <math>\mathcal{C}</math>, then <math>M_A=M(\mathcal{C})</math>, which shows that for all <math>A\in\mathcal{C}</math> and <math>B\in M(\mathcal{C})</math> we get <math>A\cap B\in M(\mathcal{C})</math>. We can now apply the same idea another time. Fix <math>B\in M(\mathcal{C})</math> and define
<math display="block">
M:=\{A\in M(\mathcal{C})\mid A\cap B\in M(\mathcal{C})\}.
</math>
Now, from above, we get that <math>M</math> is a monotone class, i.e. <math>M\subset M(\mathcal{C})</math> and thus for all <math>A,B\in M(\mathcal{C})</math> we get <math>A\cap B\in M(\mathcal{C})</math>. Hence it follows that <math>M(\mathcal{C})</math> is stable under finite intersections and is therefore a <math>\sigma</math>-Algebra.}}
{{proofcard|Corollary|cor-1|Let <math>(E,\A)</math> be a measurable space and let <math>\mu,\nu</math> be two measures on <math>(E,\mathcal{A})</math>. Moreover, assume that there exists a family of subsets <math>\mathcal{C}</math>, which is stable under finite intersections, such that <math>\sigma(\mathcal{C})=\mathcal{A}</math> and <math>\mu(A)=\nu(A)</math> for all <math>A\in\mathcal{C}</math>. Then the following hold.
<ul style{{=}}"list-style-type:lower-roman"><li>If <math>\mu(E)=\nu(E) < \infty</math>, then we get <math>\mu=\nu</math>.
</li>
<li>If there exists an increasing family <math>(E_n)_{n\in\N}</math> with <math>E_n\in\mathcal{C}</math> such that
<math display="block">
E=\bigcup_{n\in\N}E_n
</math>
and <math>\mu(E_n)=\nu(E_n) < \infty</math>, then it follows that <math>\mu=\nu</math>.
</li>
</ul>
|Let us first define the set <math>G:=\{A\in\A\mid \mu(A)=\nu(A)\}</math>. By assumption we get that <math>\mathcal{C}\subset G</math>. Moreover, we note that <math>G</math> is a monotone class. Note at first that <math>E\in G</math> by assumption since <math>\mu(E)=\nu(E)</math>. Now let <math>A,B\in G</math> such that <math>A\subset B</math> and since
<math display="block">
\mu(B\setminus A)=\mu(B)-\mu(A)=\nu(B)-\nu(A)=\nu(B\setminus A),
</math>
we get that <math>(B\setminus A)\in G</math>. Now let <math>(A_n)_{n\in\N}</math> be an increasing sequence in <math>G</math>. Then the fact
<math display="block">
\mu\left(\bigcup_{n\in\N}A_n\right)=\lim_{n\to\infty}\mu(A_n)=\lim_{n\to\infty}\nu(A_n)=\nu\left(\bigcup_{n\in\N}A_n\right)
</math>
implies that <math>\bigcup_{n\in\N}A_n\in G</math>. Moreover, since <math>G</math> is a monotone class containing <math>\mathcal{C}</math>, we get that <math>G</math> contains <math>M(\mathcal{C})</math>. On the other hand we know that <math>\mathcal{C}</math> is stable under finite intersections and therefore it follows that <math>M(\mathcal{C})=\sigma(\mathcal{C})</math> and that <math>G=\sigma(\mathcal{C})=\A</math>. Now define for all <math>n\in\N</math> and <math>A\in \A</math> the two sequences
<math display="block">
\begin{align*}
\mu_n(A)&=\mu(A\cap E_n)\\
\nu_n(A)&=\nu(A\cap E_n)
\end{align*}
</math>
Now since <math>\mu(E)=\nu(E)</math>, we get the same for the sequence elements and obtain therefore that <math>\mu_n=\nu_n</math>. Moreover, for <math>A\in \A</math>, we have
<math display="block">
\mu(A)=\lim_{n\to\infty}\mu(A\cap E_n)=\lim_{n\to\infty}\uparrow\nu(A\cap E_n)=\nu(A).
</math>
Hence we get that <math>\mu=\nu</math>.}}
{{alert-info |
There are several applications of this corollary. Let us emphasize a first one, by giving already a small introduction to the Lebesgue measure. Assume that <math>\lambda</math> is a measure on the measurable space <math>(\R,\B(\R))</math> such that <math>\lambda((a,b))=b-a</math> for <math>a < b</math> and let <math>\mathcal{C}</math> be the class of intervals <math>E_n=(-n,n)</math> for <math>n\geq 1</math>. With the corollary above, it follows that <math>\lambda</math> is unique. We will call <math>\lambda</math> the Lebesgue measure. A second application is that a finite measure on <math>(\R,\B(\R))</math> is uniquely characterized, for <math>a\in\R</math>, by the values
<math display="block">
\mu((-\infty,a])=\mu((-\infty,a)).
</math>
}}
==General references==
{{cite arXiv|last=Moshayedi|first=Nima|year=2020|title=Lectures on Probability Theory|eprint=2010.16280|class=math.PR}}
==Notes==
{{notelist}}

Latest revision as of 01:54, 8 May 2024

[math] \newcommand{\R}{\mathbb{R}} \newcommand{\A}{\mathcal{A}} \newcommand{\B}{\mathcal{B}} \newcommand{\N}{\mathbb{N}} \newcommand{\C}{\mathbb{C}} \newcommand{\Rbar}{\overline{\mathbb{R}}} \newcommand{\Bbar}{\overline{\mathcal{B}}} \newcommand{\Q}{\mathbb{Q}} \newcommand{\E}{\mathbb{E}} \newcommand{\p}{\mathbb{P}} \newcommand{\one}{\mathds{1}} \newcommand{\0}{\mathcal{O}} \newcommand{\mat}{\textnormal{Mat}} \newcommand{\sign}{\textnormal{sign}} \newcommand{\CP}{\mathcal{P}} \newcommand{\CT}{\mathcal{T}} \newcommand{\CY}{\mathcal{Y}} \newcommand{\F}{\mathcal{F}} \newcommand{\mathds}{\mathbb}[/math]

A very important notion is that of a monotone class. We will see that there are many things which can be deduced by using the monotone class lemma.

Definition (Monotone Class)

Let [math]E[/math] be some topological space and let [math]M\subset\mathcal{P}(E)[/math]. [math]M[/math] is called a monotone class if the following holds.

  • [math]E\in M[/math].
  • Let [math]A\in M[/math] and [math]B\in M[/math]. If [math]A\subset B[/math] [math]\Longrightarrow B\setminus A\in M[/math].
  • Let [math](A_n)_{n\in\N}\in M[/math]. If [math]A_n\subset A_{n+1}[/math] [math]\Longrightarrow \bigcup_{n\in\mathbb{N}}A_n \in M[/math].

A [math]\sigma[/math]-Algebra is a monotone class[a]

As for [math]\sigma[/math]-Algebras, we notice that an arbitrary intersection of monotone classes is again a monotone class. Thus, if [math]\mathcal{C}\in \mathcal{P}(E)[/math], we can define the monotone class generated by [math]\mathcal{C}[/math] as

[[math]] M(\mathcal{C})=\bigcap_{\mathcal{C}\subset M\atop M\text{mon.cl.}}M. [[/math]]

This is also by construction the smallest monotone class containing [math]\mathcal{C}[/math].

Theorem (Monotone Classes lemma)

Let [math]E[/math] be a topological space. If [math]\mathcal{C}\subset\mathcal{P}(E)[/math] is stable under finite intersection, i.e. for [math]A\in \mathcal{C}[/math] and [math]B\in\mathcal{C}\Longrightarrow A\cap B\in \mathcal{C}[/math], then

[[math]] \sigma(\mathcal{C})=M(\mathcal{C}). [[/math]]


Show Proof

It is obvious that, [math]M(\mathcal{C})\subset\sigma(\mathcal{C})[/math] since a [math]\sigma[/math]-Algebra is also a monotone class. Next we want to show that [math]M(\mathcal{C})[/math] is a [math]\sigma[/math]-Algebra to conclude that [math]\sigma(\mathcal{C})\subset M(\mathcal{C})[/math] and hence then [math]M(\mathcal{C})[/math] contains [math]\mathcal{C}[/math], i.e. [math]\sigma(\mathcal{C})[/math]. It is not difficult to see that a monotone class, which is stable under finite intersection, is a [math]\sigma[/math]-Algebra. Let us therefore show that [math]M(\mathcal{C})[/math] is stable under finite intersections. First, we fix [math]A\in\mathcal{C}[/math] and define

[[math]] M_A:=\{B\in\mathcal{C}\mid A\cap B\in M(\mathcal{C})\}. [[/math]]

Then we get that [math]\mathcal{C}\in M_A[/math] since [math]\mathcal{C}[/math] is stable under finite intersections and obviously [math]E\in M_A[/math]. We can also note that If [math]B,B'\in M_A[/math] and [math]B\subset B'[/math], with

[[math]] A\cap (B'\setminus B)=\underbrace{\underbrace{(A\cap B')}_{\in M(\mathcal{C})}\setminus \underbrace{(A\cap B)}_{\in M(\mathcal{C})}}_{\in M(\mathcal{C})} [[/math]]

then [math]B'\setminus B\in M_A[/math]. Moreover, if [math](B_n)_{n\in\N}\in M_A[/math] and [math]A\cap\bigcup_n\underbrace{(A\cap B_n)}_{\in M(\mathcal{C})}[/math] we get the implication


[[math]] \bigcup_{n\geq 1}(A\cap B_n)\in M(\mathcal{C})\Longrightarrow \bigcup_{n\geq 1}B_n\in M_A. [[/math]]

since [math](A\cap B_n)[/math] is increasing. Finally we can conclude the above facts. That means if [math]M_A[/math] is a monotone class containing [math]\mathcal{C}[/math], then [math]M_A=M(\mathcal{C})[/math], which shows that for all [math]A\in\mathcal{C}[/math] and [math]B\in M(\mathcal{C})[/math] we get [math]A\cap B\in M(\mathcal{C})[/math]. We can now apply the same idea another time. Fix [math]B\in M(\mathcal{C})[/math] and define

[[math]] M:=\{A\in M(\mathcal{C})\mid A\cap B\in M(\mathcal{C})\}. [[/math]]
Now, from above, we get that [math]M[/math] is a monotone class, i.e. [math]M\subset M(\mathcal{C})[/math] and thus for all [math]A,B\in M(\mathcal{C})[/math] we get [math]A\cap B\in M(\mathcal{C})[/math]. Hence it follows that [math]M(\mathcal{C})[/math] is stable under finite intersections and is therefore a [math]\sigma[/math]-Algebra.

Corollary

Let [math](E,\A)[/math] be a measurable space and let [math]\mu,\nu[/math] be two measures on [math](E,\mathcal{A})[/math]. Moreover, assume that there exists a family of subsets [math]\mathcal{C}[/math], which is stable under finite intersections, such that [math]\sigma(\mathcal{C})=\mathcal{A}[/math] and [math]\mu(A)=\nu(A)[/math] for all [math]A\in\mathcal{C}[/math]. Then the following hold.

  • If [math]\mu(E)=\nu(E) \lt \infty[/math], then we get [math]\mu=\nu[/math].
  • If there exists an increasing family [math](E_n)_{n\in\N}[/math] with [math]E_n\in\mathcal{C}[/math] such that
    [[math]] E=\bigcup_{n\in\N}E_n [[/math]]
    and [math]\mu(E_n)=\nu(E_n) \lt \infty[/math], then it follows that [math]\mu=\nu[/math].


Show Proof

Let us first define the set [math]G:=\{A\in\A\mid \mu(A)=\nu(A)\}[/math]. By assumption we get that [math]\mathcal{C}\subset G[/math]. Moreover, we note that [math]G[/math] is a monotone class. Note at first that [math]E\in G[/math] by assumption since [math]\mu(E)=\nu(E)[/math]. Now let [math]A,B\in G[/math] such that [math]A\subset B[/math] and since

[[math]] \mu(B\setminus A)=\mu(B)-\mu(A)=\nu(B)-\nu(A)=\nu(B\setminus A), [[/math]]
we get that [math](B\setminus A)\in G[/math]. Now let [math](A_n)_{n\in\N}[/math] be an increasing sequence in [math]G[/math]. Then the fact

[[math]] \mu\left(\bigcup_{n\in\N}A_n\right)=\lim_{n\to\infty}\mu(A_n)=\lim_{n\to\infty}\nu(A_n)=\nu\left(\bigcup_{n\in\N}A_n\right) [[/math]]
implies that [math]\bigcup_{n\in\N}A_n\in G[/math]. Moreover, since [math]G[/math] is a monotone class containing [math]\mathcal{C}[/math], we get that [math]G[/math] contains [math]M(\mathcal{C})[/math]. On the other hand we know that [math]\mathcal{C}[/math] is stable under finite intersections and therefore it follows that [math]M(\mathcal{C})=\sigma(\mathcal{C})[/math] and that [math]G=\sigma(\mathcal{C})=\A[/math]. Now define for all [math]n\in\N[/math] and [math]A\in \A[/math] the two sequences

[[math]] \begin{align*} \mu_n(A)&=\mu(A\cap E_n)\\ \nu_n(A)&=\nu(A\cap E_n) \end{align*} [[/math]]


Now since [math]\mu(E)=\nu(E)[/math], we get the same for the sequence elements and obtain therefore that [math]\mu_n=\nu_n[/math]. Moreover, for [math]A\in \A[/math], we have

[[math]] \mu(A)=\lim_{n\to\infty}\mu(A\cap E_n)=\lim_{n\to\infty}\uparrow\nu(A\cap E_n)=\nu(A). [[/math]]
Hence we get that [math]\mu=\nu[/math].

There are several applications of this corollary. Let us emphasize a first one, by giving already a small introduction to the Lebesgue measure. Assume that [math]\lambda[/math] is a measure on the measurable space [math](\R,\B(\R))[/math] such that [math]\lambda((a,b))=b-a[/math] for [math]a \lt b[/math] and let [math]\mathcal{C}[/math] be the class of intervals [math]E_n=(-n,n)[/math] for [math]n\geq 1[/math]. With the corollary above, it follows that [math]\lambda[/math] is unique. We will call [math]\lambda[/math] the Lebesgue measure. A second application is that a finite measure on [math](\R,\B(\R))[/math] is uniquely characterized, for [math]a\in\R[/math], by the values

[[math]] \mu((-\infty,a])=\mu((-\infty,a)). [[/math]]

General references

Moshayedi, Nima (2020). "Lectures on Probability Theory". arXiv:2010.16280 [math.PR].

Notes

  1. Prove that as an exercise.